Divide. Round your answer to the nearest tenth.
160 divided by 0.749 =

Answers

Answer 1
213.61815754
rounded to the nearest tenth is 213.6

Related Questions

Solve for brainliest and 20 points please

Answers

Answer:

C.

Step-by-step explanation:

540

Answer:

540

Step-by-step explanation:

you can use this equation to find the sum of angles for a shape
(n-2)180 with n being the number of sides. Since a pentagon has 5,
its 5-2= 3 then times 180 is 540

7TH GRADE MATH, BRAINLIEST WILL BE AWARDED

Answers

Answers:

Q1.a: 11g

Q1.b: 3m

Q1.3: [tex]-k^{2}[/tex]

Q2.a:  8x + 11y

Q2.b: 2a + b - 2c

Q3.a: 5q + 7

Q3.b: 6r + 3s + 7t

Step-by-step explanation:

Q1) simplify each of these expressions:

a. [tex]4g + 6g + g[/tex]

= g(4 + 6 + 1)

= 11g

____________

b. [tex]4m - m[/tex]

= m(4 - 1)

= 3m

____________

c. [tex]5k^{2} -4k^{2} -2k^{2}[/tex]

= [tex]k^{2} (5 - 4 - 2)\\[/tex]

= [tex]-1k^{2}[/tex]

= [tex]-k^{2}[/tex]

__________________________________

Q2) copy and complete the workings to simplify these expressions:

a. [tex]5x + 3x+6y+5y\\[/tex]

= x(5 + 3) + y(6 + 5)

= 8x + 11y

____________

b.[tex]2a+4b+6c-3b-8c[/tex]

combine terms b:

2a + b + 6c - 8c

combine terms c:

= 2a + b - 2c

__________________________________

Q3) simplify these expressions by collecting like terms:

a.[tex]9q-4q+15-8[/tex]

combine terms q:

5q + 15 - 8

combine numbers:

= 5q + 7

____________

b. [tex]8r+2s+4t-2r+s+3t[/tex]

combine terms r:

6r + 2s + 4t + s + 3t

combine terms s:

6r + 3s + 4t + 3t

combine terms t:

= 6r + 3s + 7t


Calculate the mean height of five men whose weight in kg are 60, 65, 71, 75 and 80

Answers

Answer:

Given the data set of height (inches) for 12 students, calculate the mean: 60, 64, 66, 68, 70, 71, 72, 73, 78, 71, 65, 69

Use the table to find the products od the two polynomials. Write your answer in descending order l.

Answers

Answer:

To use the given table to find product of the two polynomials.

Given that,

[tex](x^2+x-2)(4x^2-8x)[/tex]

Explanation:

Using the table, we get it as,

we get,

[tex](x^2+x-2)(4x^2-8x)=4x^4-8x^3+4x^3-8x^2-8x^2+16x[/tex][tex]=4x^4-4x^3-16x^2+16[/tex]

we get,

[tex](x^2+x-2)(4x^2-8x)=4x^4-4x^3-16x^2+16[/tex]

Answer is:

[tex](x^2+x-2)(4x^2-8x)=4x^4-4x^3-16x^2+16[/tex]

The equation y = - 50x + 2000 represents thenumber of people attending a movie where xrepresents the number of weeks since the movierelease and y represents the number of people inthe theater.In the equation, what does the y-interceptindicate?A. Two thousand people attend the theater duringthe week of a movie's release.B. Every week 50 less people attend the movietheater.C. During a movie's release week, the movietheater will make $2,000.D. Every week the movie is open, the movie theaterloses $50.

Answers

two thousand people attend the theater during the week of a movie's release (option A)

Explanation:

The equation:

y = -50x + 2000

y = number of people in the theater

x = number of weeks since the movie release

2000 will be the number of people in the theater at the begining of the movie release

y-intercept of an equation represent the value of y when x is zero

The 2000 is the number of people in the theater at week zero.

Hence, y-intercept indicate two thousand people attend the theater during the week of a movie's release (option A)


The midpoint of AB is at ( – 5, 4). If A = (0, – 9), find B.

Answers

Point of B is ( -10 , 0) is midpoint of AB .

What is midpoint of a segment?

The midpoint of a vertical line is known as the midpoint in geometry. It is the centroid of the segment and of the ends, and it is equal to the distance from both of them. It cuts the section in half.A midpoint is the position that is in the middle or center of a line connecting two points, often referred to as endpoints. The midpoint formula can be used to determine the other midpoint given one endpoint and a midpoint.

The midpoint of AB =  ( – 5, 4) = ( x ,y)

A = (0, – 9) ⇒ ( x₁ , y₁)

Let B = ( x₂, y₂ )

midpoint of AB ⇒ x =  x₁ + x₂/2 ,  y = y₁ + y₂/2

                              -5 = 0 + x₂/2  , -9 = -9 + y₂/2

                              x₂ = -10  , y₂ = 0

point of B is ( -10 , 0)

Learn more about midpoint of a segment

brainly.com/question/4747771

#SPJ13

First person to help gets brainliest!!

Answers

What is the solution to the given equation?

x= 20

How is the equation in the bar diagram solved?

3.5x + 12.5x =320

16x = 320

x = 20

What is a bar diagram?

A bar graph or chart displays categorical data using rectangular bars with heights or lengths proportional to the values they represent.Both vertical and horizontal plots of the bars are possible. Column charts are another name for vertical bar charts. Comparisons between distinct categories are shown in a bar graph. The chart's two axes, one for measured value and the other for the specified categories under comparison, are shown. In certain bar graphs, clusters of multiple bars that represent the values of multiple measured variables are clustered together.

To learn more about solving equations, refer:

https://brainly.com/question/25678139

#SPJ13

Suppose a diver jumps from a ledge that is 115 feet above the ocean and the initial upward velocity is 8 feet per second. The vertical motion of the diver can be modeled by the function h = −t^2 + 8t + 115. How long will it take until the diver enters the water? How do you know?

Answers

The given function of the height of the diver is

[tex]h=-t^2+8t+115[/tex]

h is the height in feet

t is the time in seconds

To find the time for the whole motion equate h by 0, as when the diver inter the water his jumper height will be zero. (The surface of the water is the initial position)

[tex]0=-t^2+8t+115[/tex]

Switch the 2 sides and change all signs to opposite

[tex]t^2-8t-115=0[/tex]

Now, we have a quadratic equation, then we will use the calculator to find the values of t

[tex]\begin{gathered} t=15.44552314 \\ \\ t=-7.445523142 \end{gathered}[/tex]

Since time can NOT be a negative value, then we will ignore the 2nd value of t

The answer should be about 15.44 seconds to the nearest 2 decimal place

A cone has a volume of 3014.4 cubic inches and a radius of 12 inches.what is the height?

Answers

Answer:

h = 20inches

Explanations:

The formula for calculating the volume of a cone is expressed as:

[tex]V=\frac{1}{3}\pi r^2h[/tex]

where:

r is the radius

h is the height

Given the following

r = 12 inches

V = 3014.4 cubic inches

Substitute to determine the height

[tex]\begin{gathered} 3014.4=\frac{1}{3}\times3.14\times12^2\times h \\ 3014.4\times3=452.16h \\ h=\frac{9043.2}{452.16} \\ h=20inches \end{gathered}[/tex]

Hence the height of the cone will be 20inches

The probability of a student eating at the cafeteria and a student living off campus is 0.07, and the probability of a student eating at the cafeteria given that the student lives off campus is 0.20, what is the probability of a student living off campus?

Answers

The probability of a student living off campus is 0.35.

What is the probability?

It should be noted that probability simply means the likelihood that something will occur.

Remember the multiplication rule for conditional probability: P(B AND A)=P(B/A)P(A)

Rearranging, we find that P(A)=P(B AND A)P(B/A)

So if we think of A= the event a student lives off campus and B = event a student eats at the cafeteria, then we can plug in the known information to find:

P(A)=0.07 / 0.20

= 0.35

Learn more about probability on:

https://brainly.com/question/25870256

#SPJ1

Keisha is making potato casserole for a party. she needs 3/2 of a potato per guest. how many potatoes will she need for 15 guest

Answers

Keisha needs 3/2 of potato per guest ( i.e one guest is equivalent to 3/2 potato)

Thus, for 15 guests

she needs;

[tex]\begin{gathered} 15\text{ }\times\frac{3}{2} \\ \frac{15\times3}{2}=\frac{45}{2} \\ 22\frac{1}{2} \end{gathered}[/tex]

Keisha needs 45/2 potatoes for 15 guests

Writing out and solving inequalities a number divided by three less two is at most two

Answers

The inequlaity is ;

[tex]\begin{gathered} \frac{x}{3\text{ }}\text{ - 2}\leq2 \\ \end{gathered}[/tex]

And the solution is;

[tex]x\text{ }\leq\text{ 12}[/tex]

Let the number be x

The number divided by 3

That is x/3

less two means minus 2

x/3 - 2

Is at most 2 means it is less than or equal to 2

so the inequality is;

[tex]\begin{gathered} \frac{x}{3}\text{ - 2}\leq\text{ 2} \\ \\ \text{Solving the inequality, we have} \\ \\ \frac{x}{3}\text{ }\leq\text{ 2 + 2} \\ \\ \frac{x}{3}\text{ }\leq\text{ 4} \\ \\ x\text{ }\leq\text{ 3 }\times\text{ 4 } \\ x\text{ }\leq\text{ 12} \end{gathered}[/tex]

A space shuttle is moving in a straight line and is traveling at a constant speed. It takes 3 hours to get from A to B and 1 hour to get from B to C. Relative to a suitable set of axes, A is the point (4,-1,7) and B is the point (16,-10,10). Find the coordinates of C.

Answers

We will solve as follows:

First: We will use the i, j, k vector notation to describe each point as a vector:

[tex]A=4i-j+7k[/tex][tex]B=16i-10j+10k[/tex]

Now, we have that the time it takes to get from A to B is 3 hours (t1). And the time it takes from B to C is 1 hour (t2).

Second: We determine the speed from A to B:

[tex]v=\frac{B-A}{t_1}\Rightarrow v=\frac{(16-4)i+(-10+1)j+(10-7)k}{3}[/tex][tex]\Rightarrow v=\frac{12i-9j+3k}{3}\Rightarrow v=4i-3j+k[/tex]

Third: We now determine the value of C:

[tex]C=B+vt_2\Rightarrow C=(16+4)i+(-10-3)j+(10+1)k[/tex][tex]\Rightarrow C=20i-13j+11k[/tex]

So, we would have that the coordinates of C are:

[tex]C=(20,-13,11)[/tex]

what is the arc measure of ct in radians? what is the arc length in feet?

Answers

We will determine the arch lenght (In radians) as follows:

*First: We transform the measure of the angle from degrees to radians, that is:

[tex]\theta=80\cdot\frac{\pi}{180}\Rightarrow\theta=\frac{4\pi}{9}[/tex]

*Second: We find the arc length:

[tex]s=r\theta\Rightarrow s=(13.2)(\frac{4\pi}{9})[/tex][tex]\Rightarrow s=\frac{88}{15}\pi[/tex]

So, the arc length for CT is 88pi/15 radians.

*Third: We find its measure in feet:

[tex]s\approx18.43[/tex]

So, the arch length for CT in feet is approximately 18.43 feet.

Given: GC bisects FGH. Determine the missing measurea. m

Answers

a. Since GC bisects FGH and the angle FGH = 122°, we know that the angle FGC = 122/2 = 61°

b. Since GC bisects FGH and the angle CGH = 42°, we know that the angle FGH = 2*42 = 84°

Can you please also give all forms of the end behavior such as ups/downs, as_,_ , and limits #25

Answers

[tex]f(x)=\frac{x}{2x+1}[/tex]

The given function is a rational function

We will find the zeros of the denominator

[tex]\begin{gathered} 2x+1=0 \\ 2x=-1 \\ x=\frac{-1}{2} \end{gathered}[/tex]

The end behavior of the function will be as follows:

[tex]\begin{gathered} x\rightarrow(-\frac{1}{2})^-;f(x)\rightarrow\infty \\ x\rightarrow(-\frac{1}{2})^+;f(x)\rightarrow-\infty \\ x\rightarrow\infty;f(x)\rightarrow\frac{1}{2} \\ x\rightarrow-\infty;f(x)\rightarrow\frac{1}{2} \end{gathered}[/tex]

The graph of the function is as follows:

Create a data set that has 4 values with a mode of 3, a median of 6, and a meanof 10

Answers

We are to create a data set of 4

that is a, b, c, d

mode means a number that occur the most

Median means the middle number

Mean means the average number

in the set, we must have a mode of 3

3, 3,

to get a median of 6, we need a number when we add to three and divide by 2 will give us 6

the number is 9

3 + 9 / 2

12/ 2 =

3, 3 , 9, d

the last number is d and it is unknown

since means is 10

mean = summation of all the numbers in the data set / the total number

the total number is 4

mean = 10

10 = 3 + 3 + 9 + d / 4

cross multiplication

10 x 4 = 3 + 3 + 9 + d

40 = 6 + 9 + d

40 = 15 + d

isolate d

d = 40 - 15

d = 25

therefore, the data set with a mode of 3, median of 6 and a mean of 10 are

3 , 3 , 9, 25

After making your 20th payment of $524.50 on your car loan, you wanted to find out how much is left of your original 5 years loan at 6.2% compounded monthly of $27,000.00. What is the amount of the remaining balance of your car loan?

Answers

The amount of the remaining balance of the car loan is $26293.1103.

Given,

The initial loan amount, P = 27000

The rate of interest, r = 6.2%

Time period, t = 5 years

Interest is compounded monthly.

The 20th payment of the loan = $524.50

We have to find the  remaining balance of the car loan.

Here,

Amount, A = P[1 + r/n]^nt

Where,

A is the total amount = principal amount + compound interest

P is the principal amount

r is the rate of interest

n is the number of times compounded

t is the time period

So,

A = P[1 + r/n]^nt

A = 27000 × [1 + 6.2/100/12]^12 × 5

A = 27000 × [12.062/12]^60

A = 27000 × [1.005166]^60

A = 27000 × 1.362

A = 36783.1103

The total amount should be paid after 60 months is $36783.1103

Now,

The 20th payment = $524.50

That is,

524.20 × 20 = 10490

$10490 is paid and the balance amount;

36783.1103 - 10490 = 26293.1103

Therefore,

The amount of the remaining balance of the car loan is $26293.1103.

Learn more about compound interest here;

https://brainly.com/question/14746481

#SPJ1

How many positive real zeroes does f(x) = x⁵ - 4x³ + 7x² + 3x - 5 have?

Answers

Given -

f(x) = x⁵ - 4x³ + 7x² + 3x - 5

To Find -

How many positive real zeroes does f(x) have =?

Step-by-Step Explanation -

We have

f(x) = x⁵ - 4x³ + 7x² + 3x - 5

where the signs of coefficients are:

+ - + + -

We can see there are three changes in the sign in f(x).

So, from

Descarte's rule,

there are either 3 0r 1 positive real roots

Now,

f(-x) = -x⁵ + 4x³ + 7x² - 3x - 5

Signs: - + + - -

So, here f(-x) has two sign changes.

From this we can conclude there is at least 1 real root and either 4, 2 or 0 imaginary roots.

Final Answer -

Positive real zeroes f(x) have =

Minimum = 1

Maximum = 3

What would be the first step when solving using the substitution method?
2x + 3y = -12

x = 3y + 3

Answers

The first step of substitution method for given system is put x = 3y +3 in place of x into the equation 2x+3y=-12 .

What is the method of substitution for system of equation in two variables ?

The system of the equations in two variables x and y consists of two equation of the form a₁ x + b₁ y = c₁

and a₂ x + b₂ y = c₂

where a₁, a₂, b₁, b₂ , c₁, c₂ are constants

For substitution method, first expression of one variable is found from one equation and then put this expression in another equation to create a linear equation in second variable. Solve the linear equation in one variable for second variable and then put the value of second variable in first equation to solve for first variable

Given system of equations is :

2x + 3y = -12       .....(1)

x = 3y + 3          .....(2)

So the first step will be take expression for one variable from one equation and put it into another equation.

Take x = 3y +3 which is an expression for x from equation (2),

Then put it into equation (1)

⇒ 2 (3y +3 ) + 3 y = -12 , This is the first step of substitution method for this variable

Also, Learn more about the system of equation in two variables from the link below:

brainly.com/question/24085666

#SPJ1

Look at the figure below:If siny - and tan yo =9-what is the value of cos yº? O cos y 90cos yOcos yO cos y® - 90

Answers

The trigonometric functions tangent, sine and cosine are related by the following equation:

[tex]\tan \theta=\frac{\sin \theta}{\cos \theta}[/tex]

In this case, the angle is y°, so:

[tex]\tan y\degree=\frac{\sin y\degree}{\cos y\degree}[/tex]

Since we want the cosine, we can solve for it ans substitute the given tangent and sine:

[tex]\cos y\degree=\frac{\sin y\degree}{\tan y\degree}=\frac{\frac{9}{c}}{\frac{9}{d}}=\frac{9}{c}\cdot\frac{d}{9}=\frac{d}{c}[/tex]

This corresponds to the third alternative.

If the function h(x)=(x+7)9 is expressed in the form f∘g with f(x)=x9, find the function g(x).

Answers

Answer:

g(x) = x + 7

Step-by-step explanation:

f(x) = x^9

g(x) = x + 7

f(g(x)) = (x + 7)^9

- 1/2 + 5/9 someone help

Answers

everything u need id in the picture

One weekend 5,780 people saw a new movie at (7 different theaters. Each theater sold tickets at ($7.50 a piece. If each theater received the same number of moviegoers, how much did each theater make?

Answers

Each theater sold tickets at ($7.50 a piece, if each theater received the same number of moviegoers, each theater make $6192.85 using arithmetic operations.

What are arithmetic operations?

The four basic operations of arithmetic can be used to add, subtract, multiply, or divide two or more quantities. They cover topics like the study of numbers and the order of operations, which are relevant to all other areas of mathematics including algebra, data processing, and geometry. Without applying the laws of arithmetic operations, we are unable to solve the issue. The four fundamental rules of mathematics are addition, subtraction, multiplication, and division.

Let the amount of each theater get be x,

No. of theaters are = 7

Cost of each ticket = $7.50

No. of people = 5780

Then we can say,

7x = 5780 ×7.50

x = [tex]\frac{5780\times7.50}{7}[/tex]

= $6192.85

To know more about arithmetic operations, visit:

https://brainly.com/question/20595275

#SPJ13

What is the place value of the 6-digit in the number 205.876?

Answers

The place value of 6 in 205.876 is 6 thousandth

1936 divided by 8 in long division

Answers

The value of 1936 divided by 8 in long division is 242.

What is long division?

Long Division is a method for dividing large numbers that divides the task into numerous phases that follow a sequence. Just like in conventional division problems, the dividend is divided by the divisor, yielding the quotient and, in some cases, a remainder.

The steps are:

Step 1: Take the first digit of the dividend from the left.

Step 2: Then divide it by the divisor and write the answer on top as the quotient.

Step 3: Subtract the result from the digit and write the difference below.

Step 4: Bring down the next digit of the dividend

Using the above step the division is 242.

Learn more about division in:

https://brainly.com/question/27601809

#SPJ1

this. this is sucking my soul away.

Answers

Check the picture below.

let's recall that twin sides stemming from a common vertex, make twin angles at the base.

Write a function to model the geometric sequence in the table. n: (1 2 3 4 5)a: (75 15 3 3/5 3/25)a) f(n) = 75 (1/5) nb) f(n) = 75 (1/5) n-1c) f(n) = 1/5 (75) nd) f(n) = 1/5 (75) n-1

Answers

First let's find the ratio of the sequence, by dividing one term by the term before:

[tex]\begin{gathered} \text{second term: 15} \\ \text{first term: 75} \\ ratio=\frac{15}{75}=\frac{1}{5} \end{gathered}[/tex]

So the ratio is 1/5 and the first term is 75.

Now, we can use the following formula for the nth term of a geometric sequence:

[tex]a_n=a_1\cdot q^{n-1}[/tex]

Where q is the ratio and a1 is the first term. So we have:

[tex]a_n=75(\frac{1}{5})^{n-1}[/tex]

Substituting an by the function f(n), we have:

[tex]f(n)=75(\frac{1}{5})^{n-1}[/tex]

So the correct option is b)

A cake of mass 550 g has three dry ingredients: flour, sugar and raisins.
There is twice as much flour as sugar and one and a half times as much
sugar as raisins. How much flour is there?

Answers

Answer:

Let flour, sugar, raisins be 3x, 1.5x, x grams respectively.

Total Mass = 550g 3x+1.5x + x = 550g

5.5x = 550 x = 100g

Flour = 3x= 3 x 100 = 300g

Hence, flour in mixture is 300g

Help me asp please!!

Answers

It is an ongoing function. A function from a set X to a set Y assigns exactly one element of Y to each element of X. The set X is known as the function's domain, and the set Y is known as the function's codomain.

What is function?In mathematics, a function is an expression, rule, or law that defines a connection between one variable (the independent variable) and another variable (the dependent variable).A function is defined as a relationship between a set of inputs and one output for each. A function is an input-output relationship in which each input corresponds to exactly one output.Every function has a domain and a codomain, as well as a range. In general, a function is denoted by f(x), where x is the input. A function is a type of rule that produces one output for one input. Alex Federspiel provided the image. y=x2 is an example of this.

To learn more about function, refer to:

https://brainly.com/question/11624077

#SPJ13

Other Questions
David left $300 in an account that earns 5% on the money he originally deposited. When he collected his interest, he had earned $60 in interest. How long did he leave the money in the account? PLS HELP METhe following table shows the distance from the bus stop as a function of time:Time (in minutes) Distance (in meters) (x) f(x) 0 45 3 40 6 35 9 30 12 25Find and interpret the meaning of the x-intercept in this scenario. (4 points)(45, 0); the distance away from the bus stop(27, 0); the time it takes to reach the bus stop(45, 0); the time it takes to reach the bus stop(27, 0); the distance away from the bus stop Find an equivalent statement for the following absolute value equation |x - 5] = 1x +6 if g(x) = 1/2|x|-6 and h(x)=x-4 Which expression is equivalent to h(g(x))? 1/2|x| - 21/2|x-4| -61/2|x|-101/2|x-10| Abu Bakr, Muhammad's father-in-law, succeeded him upon his death as the spiritual and political leader of the Islamic world. options: True False for several years in a row, the east coast cities of the nation suffered extremely cold winters. demand for home heating oil was very inelastic. how did the elasticity of supply impact the changes in price that resulted from the increased demand? here is no right or wrong answer when determining a theme if there is evidence to support it. true or false Which of the following statements correctly explains the relationship between FM and AM radio waves?A.FM radio waves carry less energy than AM radio waves. This is supported by the fact that FM radio waves have a shorter wavelength compared to AM radio waves.B.FM radio waves carry less energy than AM radio waves. This is supported by the fact that FM radio waves have a longer wavelength compared to AM radio waves.C.FM radio waves carry more energy than AM radio waves. This is supported by the fact that FM radio waves have a shorter wavelength compared to AM radio waves.D.FM radio waves carry more energy than AM radio waves. This is supported by the fact that FM radio waves have a longer wavelength whats the gcf of 96 and 40 The personal assets and liabilities of a plumber are listed below.What is the total value of the plumber's liabilities?$149,367$167,925$241,231$273,881 Remy is applying for a job where the employer asks that a cover letter and rsum be submitted. Remy should:a.Attach letters of recommendation alsob.Send the cover letter first, and the rsum laterc.Submit only the cover letter and rsumd.Send the cover letter as an attachment Question 2 of 25The coach of a college basketball team categorizes the number of pointsscored by the team in each game, with the categories being 51 - 57, 58 - 64,65 - 71, and 72 or more. If the following data represent the numbers ofpoints scored in the last 10 games, how many games were in the 51 - 57category?51, 70, 63, 59, 56, 75, 60, 55, 67, 64O A1B. 2O c. 4O D. 3 Explain how travelling to a ski resort to go skiing could influence travel time the piano played a crucial role in 19th-century musical culture.a. Trueb. False Refer to Passage 2 "War of the Worlds".Which of the following best states the main (central) idea of this passage?The main idea of this article is...To define the number of objects in our solar systemBoth Pluto and 2003 UB313 should be considered planetsThat scientists need to encourage children that science is excitingThat planets are incredibly difficult to find and define what is the name of a protocol that lets users share files over a network? 1 point apache dns cups nfs 2. question 2 which two of these methods could you use to set up printing in an organization? The intent of the Indian Removal Act was to The magnitude of the electric force between two particles depends on whichfactors? A. Charge and densityOB. Charge and distanceOC. Mass and chargeOD. Mass and distance Carrots costs $0.99 per pound. Broccoli costs $1.50 per pound. Jack buys 3.5 pounds of carrots and 1.25 pounds of broccoli. What is Jacks total bill? Please help me im getting stressed with solution